Jump to content

yeutoan11's Content

There have been 332 items by yeutoan11 (Search limited from 17-05-2020)



Sort by                Order  

#368830 Cho x,y,z >0. CMR:$\frac{xyz}{(1+3x)(x+8y)(y+9z)...

Posted by yeutoan11 on 11-11-2012 - 20:51 in Bất đẳng thức và cực trị

Cho x,y,z >0. CMR:$\frac{xyz}{(1+3x)(x+8y)(y+9z)(z+6)}\leq \frac{1}{7^{4}}$
Khi nào dấu bằng xảy ra?

BĐT tương đương
$(1+3x)(1+\frac{8y}{x})(1+\frac{9z}{y})(1+\frac{6}{z})\geq 7^4$
Holder trực tiếp có ĐPCM



#368020 ĐỀ CHỌN HSG THÀNH PHỐ HÀ NỘI VÒNG 2 NĂM HỌC 2012-2013

Posted by yeutoan11 on 08-11-2012 - 23:01 in Thi HSG cấp Tỉnh, Thành phố. Olympic 30-4. Đề thi và kiểm tra đội tuyển các cấp.

Giả sử tồn tại $2$ số $3^x;3^y$ có cùng số dư khi chia cho $83$
$\Rightarrow 3^x-3^y\vdots 83\Rightarrow 3^y(3^{x-y}-1)$
$\Rightarrow \exists k\in [1;40]:3^k\equiv 1$ (mod$83$)
Theo định lí $Fermat$ nhỏ,ta có:$3^{82}\equiv 1$ (mod $83$)
Vậy,theo định lí cấp của phần tử,để $3^k\equiv 1$ (mod $83$) thì $k|82$
$\Rightarrow k=\left \{ 1;2;41 \right \}$
Nhưng vì $k<41$ nên $\Rightarrow k=\left \{ 1;2 \right \}$
Thử lại thấy vô lí.Vậy điều giả sử là sai.
Vậy ta có $Q.E.D$

Có vẻ phải như thế này : Gọi $a$ là số nguyên dương nhỏ nhất thỏa : $3^a - 1\vdots 83$
Lúc ấy ta có : $a|x-y$ Và $a|82$ (bài bạn chưa đảm bảo tính nhỏ nhất của $k$ )
$\Rightarrow a\in\left \{ 1,2,41 \right \}$
$a=1,2$ không thỏa $\Rightarrow a=41$ Dễ thấy vô lí vì $x-y \le 40$
Vậy ta có ĐPCM



#367697 Tìm nghiệm nguyên của phương trình ${x^2} + {y^2} =...

Posted by yeutoan11 on 07-11-2012 - 18:38 in Phương trình, hệ phương trình và bất phương trình

Tìm nghiệm nguyên của phương trình ${x^2} + {y^2} = 9900$

Dùng liên tiếp bổ đề với bổ đề $p$ nguyên tố có dạng $4k+3$ Mà $a^2 + b^2 \vdots p$ Thì $a,b \vdots p$ TH này $p =3 ; p =11$



#367686 Chứng minh rằng số a là 1 hợp số lẻ, không chia hết cho $3$ và...

Posted by yeutoan11 on 07-11-2012 - 17:59 in Số học

Giải như sau:
Ta có $a=\dfrac{(3^p)^2-1}{8}=\dfrac{(3^p-1)}{2}.\dfrac{3^p+1}{4}$ hiển nhiên là hợp số vì dễ cm $\dfrac{3^p-1}{2}$ và $\dfrac{3^p+1}{2}$ là số nguyên $>1$
Ta có $a-1=\dfrac{9^p-9}{8}=9.(9^{p-2}+9^{p-3}+...+9+1)$
Dễ thấy $9^{p-2}+9^{p-3}+...+9+1$ chẵn, như vậy suy ra $a-1$ chẵn
Mặt khác $a-1=\dfrac{9^p-9}{8} \vdots p$ do theo Fermat nhỏ và $gcd(p,2)=1$ nên suy ra $a-1 \vdots p$
Như vậy $a-1 \vdots 2p$
Do đó $3^{a-1}-1 \vdots 3^{2p}-1 \Rightarrow 3^{a-1}-1 \vdots 9^p-1 \Rightarrow 3^{a-1}-1 \vdots \dfrac{9^p-1}{8}=a$ đpcm


Bài này hình như có liên quan đến bổ đề : Nếu $a$ có dạng : $\frac{b^{2p}-1}{b^2-1}$
Với $p$nguyên tố $(b,p)=1$ thì $b^{a-1} -1\vdots a$



#367467 Tìm nghiệm nguyên dương của PT: $y^2=x^2(x^2+x+1)+(x+1)^2$

Posted by yeutoan11 on 06-11-2012 - 16:45 in Số học

Tìm nghiệm nguyên dương của PT:
$y^2=x^2(x^2+x+1)+(x+1)^2$

Dạng quen thuộc dùng kẹp :
$(2x^2+x+1)^2 < (2y)^2 < (2x^2+x+3)^2$
$\Rightarrow 4y^2=(2x^2+x+2)^2$
Thay $y$ theo giả thiết có nghiệm



#367062 $\frac{a^{2}}{2a^{2}+(b+c)^...

Posted by yeutoan11 on 04-11-2012 - 18:24 in Bất đẳng thức và cực trị

Ta có : $(b+c)^2\leq 2\left ( b^2+c^2 \right )$
$\frac{a^2}{2a^2+(b+c)^2}\geq \frac{a^2}{2\left ( a^2+b^2+c^2 \right )}$
Tương tự cho 2 bđt còn lại rồi cộng lại với nhau ta được ĐCCM?

Mới có $\frac{1}{2}$



#365229 $$\left(\frac{a}{a-b}\right)^2+...

Posted by yeutoan11 on 27-10-2012 - 15:04 in Bất đẳng thức và cực trị

1) Từ điều kiện $\Rightarrow (a-b)(b-c)=\frac{ab}{2}$
Áp dụng bất đẳng thức $AM-GM$ ta có:
$$\left(\frac{a}{a-b}\right)^2+\left(\frac{2b}{b-c}\right)^2 \geq 4\left | \frac{ab}{(a-b)(b-c)} \right |=8$$
Và mặt khác
$$\left(\frac{3c}{c-a}\right)^2\geq 0$$
Từ 2 điều trên ta có điều phải chứng minh $\square$



#363874 KỲ THI THÀNH LẬP ĐỘI TUYỂN HSG LỚP 12 THPT DỰ THI QUỐC GIA TỈNH BÌNH THUẬN

Posted by yeutoan11 on 22-10-2012 - 18:42 in Thi HSG cấp Tỉnh, Thành phố. Olympic 30-4. Đề thi và kiểm tra đội tuyển các cấp.

Bài này làm không biết đúng không nữa :mellow:
Áp dụng BDT tam giác $MA+MC \geq AC,MB+MD \geq BD$
$=>MA+MB+MC+MD \geq AC+BD$
$=>min(MA+MB+MC+MD)=AC+BD$ đẳng thức xảy ra khi M là giao điểm của AC và BD

Có lẽ phải dùng thêm dirichlê để nói tồn tại 2 đường cắt nhau trong 4 điểm A,B ,C , D.
AC // BD thì ............



#363336 $$P=(a-b)(b-c)(c-a)$$

Posted by yeutoan11 on 20-10-2012 - 20:10 in Bất đẳng thức và cực trị

1 ) . Ta có : $\frac{2}{3}\sum \frac{ab+bc+ca}{(b+c)^2}=\frac{2}{3}\sum\frac{a}{b+c}+\frac{2}{3}\sum\frac{bc}{(b+c)^2}$
Vậy BĐT cần CM tương đương :
$\frac{1}{3}\sum \frac{a}{b+c}\geq \frac{2}{3}\sum\frac{bc}{(b+c)^2}$
Lại có : $\frac{2}{3}\sum\frac{bc}{(b+c)^2}\leq \frac{2}{3}\sum \frac{bc}{4bc}=\frac{1}{2}$
Và theo $Nesbit$ Ta có $VT \ge \frac{1}{2}$. Vậy có ĐPCM



#362766 bài toán bàn cờ vua

Posted by yeutoan11 on 18-10-2012 - 15:30 in Số học

dạ tức là bỏ đi 1 ô vuông nhỏ trong 64 ô vuông nhỏ trong bàn cờ ạ

____

hxthanh@: Ý tôi là em bỏ đi cả 4 đỉnh của ô vuông đó? Vì đỉnh mới quyết định đến số lượng hình chữ nhật chứ ô vuông đâu quyết định được gì?

Có lẽ là nếu bỏ đi một ô vuông thì tìm số HCN tối đa và tối thiểu có "bề mặt lấp đầy" từ các hình vuông nhỏ



#362761 CM $f(x)=(x-1)(x-2)...(x-2012)-1$ không phân tích được thành tích đ...

Posted by yeutoan11 on 18-10-2012 - 14:53 in Đại số

Chứng minh đa thức $f(x)=(x-1)(x-2)...(x-2012)-1$ không phân tích được thành tích 2 đa thức với hệ số nguyên.

Giả sử $f(x)$ phân tích được thành tích của hai đa thức có bậc dương và hệ số nguyên
$f(x)=P(x).G(x)$
Khi đó :
$f(i)=P(i).G(i)=-1 \forall i=\overline{1,2012}$
Vậy $P(i)$ và $G(i)$ nhận giá trị đối nhau $1$ và $-1$
$\Rightarrow P(i)+G(i)=0 (\forall i=\overline{1,2012})$
Đa thức $P(x)+G(x)$ có $2012$ nghiệm phân biệt nhưng theo cách đặt thì $deg(P(x)+G(x)) \le 2011$
Vậy Đa thức $P(x)+G(x)$ là đa thức $0$
$\Rightarrow P(x)= -G(x)$ $\forall x \in \mathbb{R}$
$\Rightarrow f(x) = - [P(x)]^2$
Vô lí vì $f(x)$ có hệ số cao nhất là $1$ còn $- [P(x)]^2$ có hệ số cao nhất là âm



#362133 $\frac{1}{a^2(1+a)}+\frac{1}...

Posted by yeutoan11 on 15-10-2012 - 21:11 in Bất đẳng thức và cực trị

BĐT cần CM tương đương :
$\sum \frac{bc}{a(1+a)}\geq \frac{3}{4}$
Theo $AM-GM$ ta có :
$\frac{bc}{a(1+a)}+\frac{ca}{b(1+b)}\geq \frac{2c}{\sqrt{(1+a)(1+b)}}$
Xây dựng tương tự ta được
$\sum \frac{bc}{a(1+a)}\geq \sum \frac{c}{\sqrt{(1+a)(1+b)}}\geq 2\sum \frac{c}{a+b+2}$
Lại có :
$\sum \frac{c}{a+b+2}\geq \frac{(a+b+c)^2}{2(ab+bc+ca+a+b+c)}$
$\Leftrightarrow 2\sum \frac{c}{a+b+2}\geq \frac{1}{ab+bc+ca+1}\geq \frac{1}{\dfrac{1}{3}+1}=\frac{3}{4}$
Ta có ĐPCM



#362036 $\dfrac{a}{a^2+1}+\dfrac{b}...

Posted by yeutoan11 on 15-10-2012 - 16:59 in Bất đẳng thức và cực trị

với $a;b;c \in [\dfrac{-3}{4} ; +\infty)$

$a+b+c=1$

Tìm max

$\dfrac{a}{a^2+1}+\dfrac{b}{b^2+1}+\dfrac{c}{c^2+1}$

Dự đoán $Max=\frac{9}{10}$ khi $a=b=c=\frac{1}{3}$
Ta xét :
$f(x)=\frac{x}{x^2+1}$ với $x \in [\frac{-3}{4} ;+\infty)$
Tiếp tuyến đồ thị tại điểm $x=\frac{1}{3}$ là $y=\frac{36x+3}{50}$
Hi vọng rằng : $f(x) \le \frac{36x+3}{50}$
$\Leftrightarrow \frac{(3x-1)^2(4x+3)}{50(x^2+1)}\geq 0$. Hiển nhiên đúng $\forall x \in [\frac{-3}{4};+\infty )$
Từ đây ta có $ĐPCM$



#361808 $$\sum \frac{(2a+b+c)^2}{2a^2+(b+c)^2...

Posted by yeutoan11 on 14-10-2012 - 18:52 in Bất đẳng thức và cực trị

Cách này cũng xuất hiện tương đối quen thuộc rồi :). Ý mình là tìm ra một cách phù hợp với các khái niệm trong thi ĐH thôi :D

Anh học 11 hả :icon6: . Em tưởng hàm số và tiếp tuyến là gần gũi với thi đại học @@



#361805 $$\sum \frac{(2a+b+c)^2}{2a^2+(b+c)^2...

Posted by yeutoan11 on 14-10-2012 - 18:38 in Bất đẳng thức và cực trị

Bài toán. Cho $a;b;c>0$ chứng minh rằng

$$\frac{(2a+b+c)^2}{2a^2+(b+c)^2}+\frac{(2b+a+c)^2}{2b^2+(a+c)^2}+\frac{(2c+a+b)^2}{2c^2+(a+b)^2}\leq 8$$

__

Ưu tiên cách giải phù hợp với thi đại học nhé :)

Chuẩn hoá $a+b+c=3$
BĐT viết lại như sau
$\sum \frac{(a+3)^2}{2a^2+(3-a)^2}\leq 8$
Xét $f(x)=\frac{(x+3)^2}{2x^2+(3-x)^2}$ với $x \in (0;3)$
Ta có : $f(x)\leq \frac{4}{3}(x+1)\Leftrightarrow 3(x-1)^2(4x+3)\geq 0$
Từ đây ta có $ĐPCM$
BĐT phụ tìm = cách xét tiếp tuyến của $f(x)$ tại điểm $x=1$



#361767 $2(a+b+c)-abc\leq 10$

Posted by yeutoan11 on 14-10-2012 - 17:01 in Bất đẳng thức và cực trị

bạn ơi mình có xem qua lời giải, mình thấy không hiểu chỗ chọn $t^{2}=\frac{a^{2}+b^{2}}{2}$ ??? vì sao chọn được t như vậy???

$f(x,t,t)$ thay cho $f(x,y,z)$ thì thay $y ,z = t$ nên ta có thể chọn $t = \sqrt{yz}$ hoặc $t=\frac{y+z}{2}$ hoặc
$t=f(y,z)$ sao cho khi thay $y=z = t$ thì $f(y,z)=t$ . còn chọn $t=\sqrt{\frac{y^2+z^2}{2}}$ thì do điều kiện cho $x^2 + y^2 + z^2$ Chọn thế để dồn về biến $x$ . Đây là cách nghĩ của mình



#361205 $\forall x$ mà $\left | x \right |\le 1...

Posted by yeutoan11 on 12-10-2012 - 17:19 in Bất đẳng thức - Cực trị

Cho đa thức bậc 3 : $f(x)=ax^3 + bx^2 + cx + d$
Biết rằng : $\left | f(x) \right |\le 1$ $\forall x$ thoả $\left | x \right |\leq 1$
Chứng minh rằng :
$\forall x$ mà $\left | x \right |\le 1$ thì : $\left | f'(x) \right | \leq 9$
P/S Thịnh : tại thầy luyện ghi nhầm ấy chứ =))



#360340 Sử dụng khai triển $Abel$ để chứng minh bất đẳng thức

Posted by yeutoan11 on 09-10-2012 - 13:28 in Bất đẳng thức và cực trị

Ví dụ 2.
Ch0 $0<\beta\leq y\leq x,\alpha > 0$ Và $\alpha.y+\beta.x\geq 2\alpha.\beta$.Cm:
$$\frac{1}{x^2}+\frac{1}{y^2}\leq \frac{1}{\alpha^2}+\frac{1}{\beta^2}$$
Lời giải:
Cũng tương tự bài trên.Ta xuất phát bằng phân tích:
$$\frac{1}{\alpha^2}+\frac{1}{\beta^2}=\frac{x^2}{\alpha ^2}.\frac{1}{x^2}+\frac{y^2}{\beta ^2}.\frac{1}{y^2}$$
Sau đó sử dụng khai triển $Abel$ ta có:
$$\frac{x^2}{\alpha ^2}.\frac{1}{x^2}+\frac{y^2}{\beta ^2}.\frac{1}{y^2}=\left(\frac{1}{y^2}-\frac{1}{x^2}\right)\frac{y^2}{\beta^2}+\left(\frac{x^2}{\alpha^2}+\frac{y^2}{\beta^2}\right)\frac{1}{x^2}$$
Nhưng mặt khác lại có $\alpha.y+\beta.x\geq 2\alpha.\beta$ hay $\frac{x}{\alpha}+\frac{y}{\beta}\geq 2$ nên ${x^2}{\alpha^2}+1+\frac{y^2}{\beta^2}+1\geq 2\left(\frac{x^2}{\alpha^2}+\frac{y^2}{\beta^2}\right)\geq 4\,\,\, \to \frac{x^2}{\alpha^2}+\frac{y^2}{\beta^2}\geq 2$
Vậy nên:
$$\frac{x^2}{\alpha ^2}.\frac{1}{x^2}+\frac{y^2}{\beta ^2}.\frac{1}{y^2}\geq \frac{1}{y^2}-\frac{1}{x^2}+\frac{2}{x^2}=\frac{1}{x^2}+\frac{1}{y^2}$$
Ta có điều phải chứng minh .Đẳng thức xảy ra tại $x=\alpha,y=\beta$ $\square$
Bài toán:
Ch0 các số thực dương $x,y,z$ thỏa:
$\left\{\begin{matrix} 1\leq z\leq min(x,y)\\ x+z.\sqrt{3}\geq 2\sqrt{3}\\ y\sqrt{3}+z\sqrt{10}\geq 2\sqrt{10} \end{matrix}\right.$
Tìm giá trị lớn nhất của: $D=\frac{1}{x^2}+\frac{2}{y^2}+\frac{3}{z^2}$

Bám sát vào VD 2 Ta sẽ giải bài toán như sau:
$D=\frac{1}{x^2}+\frac{2}{y^2}+\frac{3}{z^2}=\frac{1}{x^2}+\frac{1}{z^2}+2(\frac{1}{y^2}+\frac{1}{z^2})$
Ta sẽ chuyển về 2 bài toán phụ :
I) $\left\{\begin{matrix} x\ge z\ge 1\\ x+z\sqrt{3}\ge2\sqrt{3} \end{matrix}\right.$
Tìm $MaxP=\frac{1}{x^2}+\frac{1}{z^2}$
II)$\left\{\begin{matrix} y\ge z\ge 1\\ y\sqrt{3}+z\sqrt{10}\ge 2\sqrt{10} \end{matrix}\right.$
Tìm $MaxQ=\frac{1}{y^2}+\frac{1}{z^2}$
---------------------------------------------------
I): Dư đoán Max khi $z=1$ và $x=\sqrt{3}$
Ta sẽ đi CM : $\frac{1}{x^2}+\frac{1}{z^2}\le \frac{1}{3}+1$
Áp dụng theo VD 2 :
$\frac{1}{3}+1=\frac{x^2}{3}.\frac{1}{x^2}+\frac{z^2}{1}.\frac{1}{z^2}=(\frac{1}{z^2}-\frac{1}{x^2})\frac{z^2}{1}+(\frac{x^2}{3}+\frac{z^2}{1})\frac{1}{x^2}$
Lại có : $x+\sqrt{3}z \ge 2\sqrt{3}\Leftrightarrow \frac{x}{\sqrt{3}}+z\ge 2$
$\Rightarrow (1+1)(\frac{x^2}{3}+\frac{z^2}{1})\geq (\frac{x}{\sqrt{3}}+z)^2\geq 4\Leftrightarrow \frac{x^2}{3}+\frac{z^2}{1}\ge 2$ Và $z\ge 1$
Vậy :$\frac{1}{3}+1=(\frac{1}{z^2}-\frac{1}{x^2})\frac{z^2}{1}+(\frac{x^2}{3}+\frac{z^2}{1})\frac{1}{x^2}\geq \frac{1}{z^2}-\frac{1}{x^2}+\frac{2}{x^2}=\frac{1}{x^2}+\frac{1}{z^2}=Q$
II) Dự đoán Max khi $z=1$ và $y=\frac{\sqrt{10}}{\sqrt{3}}$
Ta sẽ CM : $\frac{1}{y^2}+\frac{1}{z^2}\leq \frac{3}{10}+1$
Áp dụng theo VD 2 :
$\frac{3}{10}+1=\frac{3y^2}{10}.\frac{1}{y^2}+\frac{z^2}{1}.\frac{1}{z^2}=(\frac{1}{z^2}-\frac{1}{y^2})\frac{z^2}{1}+(\frac{3y^2}{10}+\frac{z^2}{1})\frac{1}{y^2}$
Lại có : $y\sqrt{3}+z\sqrt{10}\ge 2\sqrt{10}\Leftrightarrow \frac{y\sqrt{3}}{\sqrt{10}}+z\geq 2$
$\Rightarrow (1+1)(\frac{3y^2}{10}+z^2)\geq (\frac{y\sqrt{3}}{\sqrt{10}}+z)^2\geq 4\Leftrightarrow \frac{3y^2}{10}+z^2\geq 2$ Và $z\ge 1$
$\Rightarrow \frac{3}{10}+1=(\frac{1}{z^2}-\frac{1}{y^2})\frac{z^2}{1}+(\frac{3y^2}{10}+\frac{z^2}{1})\frac{1}{y^2}\geq \frac{1}{z^2}-\frac{1}{y^2}+2\frac{1}{y^2}=\frac{1}{y^2}+\frac{1}{z^2}=P$
vậy :
$D=Q+2P \leq \frac{59}{15}$



#360098 Tìm Min $S=x^{\sqrt{3}} +y^{\sqrt...

Posted by yeutoan11 on 08-10-2012 - 19:09 in Bất đẳng thức - Cực trị

Cho $x,y,z$ thực dương thỏa :
$\left\{\begin{matrix} x\ge 4\\3x+4y\ge 24 \\3x+4y+6z\ge 36 \end{matrix}\right.$

Tìm Min $S=x^{\sqrt{3}} +y^{\sqrt{3}}+z^{\sqrt{3}}$



#359832 ÔN THI ĐẠI HỌC 2012

Posted by yeutoan11 on 07-10-2012 - 18:47 in Bất đẳng thức và cực trị

Bài toán 36.
Ch0 các số thực không âm $a,b,c$ thỏa mãn $ab+bc+ca=1$.Tìm giá trị nhỏ nhất của biểu thức:
$$D=\frac{1}{a+b}+\frac{1}{b+c}+\frac{1}{c+a}-\frac{1}{a+b+c}$$


36)
Thay 2 bộ : $(1,1,0)$ và $(\frac{1}{\sqrt{3}};\frac{1}{\sqrt{3}};\frac{1}{\sqrt{3}})$
Ta thấy bộ $(1,1,0)$ thì $D=2$ còn bộ $(\frac{1}{\sqrt{3}};\frac{1}{\sqrt{3}};\frac{1}{\sqrt{3}})$ thì $D=\frac{7\sqrt{3}}{6} > 2$
Vậy dự là $Min=2$
Ta CM:
$D=\frac{1}{a+b}+\frac{1}{b+c}+\frac{1}{c+a}-\frac{1}{a+b+c}\geq 2$
Nhân 2 vế $a+b+c$
$\Leftrightarrow \frac{a}{b+c}+\frac{b}{c+a}+\frac{c}{b+a}+2\geq 2(a+b+c)$
Nếu ta sử dụng BĐT $Nesbit$ thì $a=b=c$. Vậy ta sử dụng
$\frac{a}{b+c}+\frac{b}{c+a}+\frac{c}{a+b}\geq \frac{(a+b+c)^2}{2(ab+bc+ca)}=\frac{(a+b+c)^2}{2}$
Vậy ta cần CM :
$\frac{(a+b+c)^2}{2}+2\geq 2(a+b+c)$ ( đúng theo $AM-GM$)
Vậy ta có $ĐPCM$
(lưu ý : BĐT $Schwarz$ tử = 0 thì mẫu = 0)



#359403 ÔN THI ĐẠI HỌC 2012

Posted by yeutoan11 on 06-10-2012 - 14:33 in Bất đẳng thức và cực trị

37)
Đặt $x=tanA;y=tanB;z=tanC$($A,B,C$ là số đó 3 góc của 1 tam giác )
BĐT cần CM :
$CosA+CosB+2CosC \leq \dfrac{9}{4}$
Ta có :
$CosA+CosB+2CosC=2Cos\frac{A+B}{2}Cos\frac{A-B}{2}+2-4Sin^2\frac{C}{2}\leq 2Cos\frac{A+B}{2}+2-4Sin^2\frac{C}{2}=2Sin\frac{C}{2}+2-4Sin^2\frac{C}{2}$
(Vì $Cos\frac{A-B}{2}\le 1$ và $Cos\frac{A+B}{2}>0$)
Vậy ta cần CM:
$Sin\frac{C}{2}+1-2Sin^2\frac{C}{2}\le \frac{9}{8}$
$\Leftrightarrow (Sin\frac{C}{2}-\frac{1}{4})^2\ge0$(Đúng)
Vậy Ta có ĐPCM



#358991 $$\widehat{DAM}=90^{o}$$

Posted by yeutoan11 on 04-10-2012 - 23:10 in Hình học

hình 1.png

a).Ta có $\widehat{ANB}=2\widehat{ADC}$ không đôi => ĐPCM
b).gọi $P$ là giao điểm $AD$ và $BC$
Ta có $AD$ là trục đẳng phương của $(O_1)$ và $(O)$
$\Rightarrow P_{P/(O_1)}=P_{P/(O)}$(1)
$BC$ là trục đẳng phương của $(O_2)$ và $(O)$
$\Rightarrow P_{P/(O_2)}=P_{P/(O)}$(2)
(1),(2)
$\Rightarrow P_{P/(O_1)}=P_{P/(O_2)}$
$\Rightarrow $$P$ thuộc trục đẳng phương của $(O_1)$ và $(O_2)$
hay P thuộc MN
Vật MN luôn qua P cố định



#358331 Tìm Max: $\frac{a}{a+bc}+\frac{b...

Posted by yeutoan11 on 02-10-2012 - 18:23 in Bất đẳng thức và cực trị

Cho $a,b,c$ dương thỏa $a+b+c=1$
Tìm Max:
$\frac{a}{a+bc}+\frac{b}{b+ac}+\frac{\sqrt{abc}}{c+ab}$



#357052 CMR: $(\frac{4}{a^2+b^2}+1)(\frac{4...

Posted by yeutoan11 on 27-09-2012 - 20:26 in Bất đẳng thức và cực trị

Cho a,b,c là các số dương và $a^2+b^2+c^2=3$.CMR:
$(\frac{4}{a^2+b^2}+1)(\frac{4}{b^2+c^2}+1)(\frac{4}{c^2+a^2}+1)\geq 3(a+b+c)^2$

Dễ thấy $3(a+b+c)^2 \leq 27$
Đặt $\frac{1}{a^2+b^2}=x$, Tương tự có $y,z$
Ta có $\frac{1}{x} = a^2+b^2$. Tương tự với $y,z$. Suy ra $\sum\frac{1}{x} = 6$
Ta cần CM : $(4x+1)(4y+1)(4z+1) \geq 27$
$\Leftrightarrow \prod (4+\frac{1}{x})\geq \frac{27}{xyz}$
Ta có bài toán : $m+n+p=6$
CM: $\prod (4+m)\geq 27mnp$
$\Leftrightarrow 160+4(mn+np+pm)\geq 26mnp$
Có : $mn+np+pm \geq 3\sqrt[3]{m^2n^2p^2}$
Đặt $\sqrt[3]{mnp}=y$
Ta cần CM : $160 + 12y^2 \geq 26y^3$
$\Leftrightarrow (2-y)(80+40y+26y^2)\geq 0$
$\Leftrightarrow y\leq 2$
Theo $AM-GM$: $6=m+n+p \geq 3y$
$\Leftrightarrow y\leq 2$
Có $DPCM$



#356611 $\left (a^2+bc\right )\left (b^2+ca\right )\lef...

Posted by yeutoan11 on 25-09-2012 - 18:55 in Bất đẳng thức - Cực trị

Không mất tính tổng quát giả sử $c=min(a;b;c)$.Ta có:
$$(a+b)^2(b+c)^2(c+a)^2- 4(a^2+bc)(b^2+ca)(c^2+ab) -32a^2b^2c^2$$
$$=(a-b)^2(b-c)^2(c-a)^2+8abc^2(a-b)^2+4abc(a+b)(a-c)(b-c)\ge 0$$
$$\Rightarrow (a+b)^2(b+c)^2(c+a)^2\ge 4(a^2+bc)(b^2+ca)(c^2+ab) +32a^2b^2c^2$$
Lại có: $$a^2b^2c^2\ge 0$$
và $$(a+b)(b+c)(c+a)=2$$.
nên BĐT được chứng minh.
Dấu bằng xảy ra khi $a=b=1;c=0$ hoặc các hoán vị.

Có thể chỉ mình hướng làm không :@)!